Was ist der äquivalente Widerstand in diesem unendlichen Gitter aus Widerständen? [abgeschlossen]

Ich habe gesucht und konnte es auf der Website nicht finden, also hier ist es (zitiert nach dem Brief):

Was ist der äquivalente Widerstand zwischen den beiden markierten Knoten in diesem unendlichen Gitter aus idealen Ein-Ohm-Widerständen?

Nerd-Sniping

Mit Link zur Quelle .

Ich bin mir nicht sicher, ob es eine Antwort auf diese Frage gibt. Angesichts meiner mangelnden Erfahrung mit grundlegender Elektronik könnte es jedoch sogar einfach sein.

Ich habe den Titel von XKCD sofort wiedererkannt [Nerd Snipping is one of my favorites].
@MarkEichenlaub In Bezug auf Ihren zweiten Kommentar: Ich habe nicht die Zeit oder den Willen, jedes einzelne Ergebnis detailliert darzustellen, aber im Wesentlichen werden drei verschiedene Probleme angesprochen (angrenzend, diagonal und "Ritterzug" oder sogar vier, wenn Sie die allgemeine Lösung zählen ). Die obere Lösung ist über 2800 Wörter lang, geht in viele mathematische Details und löst nur das allgemeine Diagonalproblem. Ich denke, die Frage braucht immer noch eine prägnante, klare, organisierte Antwort, die leicht zu finden ist.
Es ist ein Duplikat, aber zum Schließen ist es zu spät; Ich nenne es ein "gutes Duplikat" und lasse es in Ruhe.

Antworten (2)

Nerd-Sniping !

Die Antwort ist 4 π 1 2 .

Einfache Erklärung :

Sukzessive Approximation! Ich beginne mit dem einfachsten Fall (siehe Abbildung unten) und füge immer mehr Widerstände hinzu, um zu versuchen, ein unendliches Gitter von Widerständen anzunähern.

Simulationsergebnisse

Mathematische Herleitung :

R m , m = 2 π ( 1 + 1 3 + 1 5 + 1 7 + + 1 2 m 1 )

+1, aber es wäre noch besser, die Lösung im Beitrag zu skizzieren, damit die Leute nicht auf einen Link klicken müssen, um zu sehen, wie es gemacht wird.
Das Zeug zu diesem Mathe-Link ist ziemlich kompliziert ... Zu viel für bloße unmenschliche Lebensformen wie mich.
Ja, ich brauchte ein paar Lesungen, um herauszufinden, wie es gemacht wurde. (Das macht es "lustig" :-P)
@David Zaslavsky: In diesem speziellen Fall bin ich wahrscheinlich viel zufriedener mit der numerischen Berechnung als mit der genauen Herleitung.
@Sklivvz: Unabhängig davon sollten wir eine Erklärung und nicht nur einen Link in der Antwort haben. (Für Ihre Antwort, so wie sie ist, war ich vielleicht zu schnell, um auf die Upvote-Schaltfläche zu klicken.)
-1 Das ist definitiv keine einfache Erklärung, ich möchte einfach sehen. Alles, was Sie brauchen, um dies zu lösen, ist das Ohmsche Gesetz zusammen mit dem von Kirchoff.
Erklärung != Simulation. Außerdem möchte ich die allgemeine Lösung für zwei beliebige Punkte wissen.
@kalle43 Ich kenne keine bessere Herangehensweise an das Problem als die in den Links von @Sklivvz beschriebenen. Nur weil Sie einfach wollen, heißt das nicht, dass es existiert. Wenn es so einfach ist, warum finden Sie nicht selbst eine einfache Antwort!
@Mark Ich habe bald einen elementaren Beweis, bin mir aber nicht sicher, ob ich ihn hier veröffentlichen soll.
@kalle43: Die "Simulation" ist auch eine Erklärung (viel mehr als bloße Mathematik). Wenn Sie den "inneren" Kreislauf betrachten und immer mehr Schichten hinzufügen, sehen Sie, dass ihr Beitrag immer weniger signifikant ist. In der Praxis funktioniert die Schaltung also ziemlich lokal und Widerstände, die mehr als beispielsweise 20 Schritte entfernt sind, sind unbedeutend.
@kalle: Das Kirchhoffsche Gesetz habe ich in meinem obigen Kommentar erwähnt. Sie erhalten eine unendlich dimensionale Matrix und müssen ihre Determinante berechnen. Oder Sie verwenden verschiedene Dualitäten, die Widerstandsnetzwerke mit Modellen der statistischen Physik verbinden. Trotzdem bezweifle ich sehr, dass irgendeine mögliche Methode in irgendeiner Weise einfach ist. Sie müssen auf jeden Fall irgendwann eine Fourier-Transformation oder nicht-triviale Integrale (wie im Link von Sklivvz) durchführen, um die Ergebnisse zu erhalten. Sie sagen also, dass Sie etwas Einfaches erhalten haben, das diese etablierten Methoden übertreffen kann? Ich kann nicht sagen, dass ich nicht an dir zweifle ;-)
@JOHA: Die Ränder hier sind wahrscheinlich sowieso zu klein für deinen Beweis. ;)
R m , m ist der Widerstand zwischen ( 0 , 0 ) und ( m , m ) , also auf der Diagonalen. Die Frage fragt nach R 2 , 1 , also die Aufnahme von R m , m in der Antwort ist überflüssig und sollte wahrscheinlich gelöscht werden.
Und irgendwie bekommen wir pi da rein ohne irgendwelche Kreise. Mathe ist erstaunlich.
Ich bevorzuge die 0,773-Näherung als Antwort. aber andererseits würde mich dieser Bus wahrscheinlich nie auf eine traurige Weise treffen. :(

Das ist das XKCD Nerd Sniping Problem . Es zwang mich, alles andere aufzugeben, was ich tat, um diese Antwort zu recherchieren und aufzuschreiben. Dann, Jahre später, zwang es mich, zurückzukehren und es der Klarheit halber zu bearbeiten.

Die folgende vollständige Lösung basiert auf den Links in der anderen Antwort. Aber zusätzlich zu der Darstellung dieser Informationen in einer bequemen Form habe ich auch einige bedeutende Vereinfachungen vorgenommen. Jetzt braucht es nur noch Highschool-Integration!

Die Strategie auf den Punkt gebracht ist

  1. Schreiben Sie einen Ausdruck für den Widerstand zwischen zwei beliebigen Punkten als Integral auf.

  2. Verwenden Sie Integrationstricks, um das in Schritt 1 gefundene Integral für zwei diagonal getrennte Punkte auszuwerten.

  3. Verwenden Sie eine Wiederholungsbeziehung, um alle anderen Widerstände aus den in Schritt 2 gefundenen zu bestimmen.

Das Ergebnis ist ein Ausdruck für alle Widerstände, von denen der Springerzug nur einer ist. Die Antwort darauf stellt sich heraus

4 π 1 2

Einrichten des Problems

Wir sind zwar letztlich an einem zweidimensionalen Gitter interessiert, aber von der Dimension hängt erstmal gar nichts ab. Deshalb beginnen wir mit der Einarbeitung N Dimensionen und spezialisieren sich auf N = 2 nur wenn nötig.

Beschriften Sie die Rasterpunkte mit n , ein N -Komponentenvektor mit ganzzahligen Komponenten.

Angenommen, die Spannung an jedem Punkt ist v n . Dann fließt der Strom hinein n von seinem 2 N Nachbarn ist

ich , ± ( v n ± e ich v n )

( e ich ist der Einheitsvektor entlang der ich -Richtung.)

Bestehen Sie darauf, dass eine externe Quelle einen Verstärker hineinpumpt 0 und aus a . Aktuelle Konservierung bei n gibt

(1) ich , ± ( v n ± e ich v n ) = δ n + δ n a

( δ n gleich 1 wenn n = 0 und 0 Andernfalls.)

Lösen Sie diese Gleichung für v n wird uns unsere Antwort geben. In der Tat, der Widerstand zwischen 0 und a wird einfach sein

R a = v 0 v a

Leider gibt es unendlich viele Lösungen für v n , und ihre Ergebnisse für R a nicht dafür! Dies liegt daran, dass die Frage keine Randbedingungen im Unendlichen angibt. Je nachdem, wie wir sie auswählen, können wir einen beliebigen Wert erzielen R a wir mögen! Es wird sich herausstellen, dass es eine einzigartige vernünftige Wahl gibt, aber lassen Sie uns dieses Problem vorerst vollständig vergessen und einfach eine Lösung finden.

Lösung durch Fourier-Transformation

Um unsere Gleichung zu lösen für v n , suchen wir nach einer Green-Funktion G n eine ähnliche Gleichung erfüllen:

(2) ich , ± ( G n ± e ich G n ) = δ n

Eine Lösung für (1) wird dann sein

v n = G n + G n a

Finden G n , nehmen Sie (aus heiterem Himmel) an, dass es als dargestellt werden kann

G n = 0 2 π d N k ( 2 π ) N ( e ich k n 1 ) g ( k )

für eine unbekannte Funktion g ( k ) . Dann beachten Sie, dass die beiden Seiten von (2) kann geschrieben werden als

ich , ± ( G n ± e ich G n ) = 0 2 π d N k ( 2 π ) N e ich k n ( ich , ± e ± ich k ich 2 N ) g ( k ) δ n = 0 2 π d N k ( 2 π ) N e ich k n

wir sehen (2) kann durch Auswahl gelöst werden

g ( k ) = 1 ich , ± e ± k ich 2 N

was zur Green-Funktion führt

G n = 1 2 0 2 π d N k ( 2 π ) N cos ( k n ) 1 ich cos ( k ich ) N

Übrigens das komische 1 im Zähler scheint nicht viel anderes zu tun als zu verschieben G n durch das Hinzufügen einer Gesamtkonstante, sodass Sie sich vielleicht fragen, was es dort tut. Die Antwort ist, dass es technisch erforderlich ist, um das Integral endlich zu machen, aber ansonsten spielt es keine Rolle, da es aus der Antwort herausgestrichen wird.

Die endgültige Antwort für den Widerstand lautet also

R a = v 0 v a = 2 ( G a G 0 ) = 0 2 π d N k ( 2 π ) N 1 cos ( k a ) N ich cos ( k ich )

Warum ist das die richtige Antwort?

(Ab diesem Zeitpunkt N = 2 .)

Ich habe vorhin gesagt, dass es unendlich viele Lösungen für gibt v n . Aber der oben ist besonders, weil auf große Entfernungen r Vom Ursprung her verhalten sich die Spannungen und Ströme wie

v = Ö ( 1 / r ) ich = Ö ( 1 / r 2 )

Ein Standardsatz (Eindeutigkeit der Lösungen der Laplace-Gleichung) besagt, dass es nur eine Lösung geben kann, die diese Bedingung erfüllt. Unsere Lösung ist also die einzigartige, bei der der geringstmögliche Strom bei unendlich und mit fließt v = 0 . Und selbst wenn die Frage nicht danach gefragt hat, ist es offensichtlich das einzig Vernünftige, was man fragen kann.

Oder ist es? Vielleicht würden Sie es vorziehen, das Problem zu definieren, indem Sie an einem endlichen Gitter arbeiten und die eindeutige Lösung für finden v n dort, dann versuchen, eine Art Grenze zu nehmen, wenn die Gittergröße ins Unendliche geht. Allerdings kann man argumentieren, dass die v n erhalten von einer Größe- L Gitter sollte zu unserem konvergieren v n mit einem Bestellfehler 1 / L . Das Endergebnis ist also dasselbe.

Der Diagonalfall

Es ergibt sich das Integral für R n , m ist schwierig zu tun, wann n m , aber viel einfacher zu tun, wenn n = m . Daher werden wir uns zuerst mit diesem Fall befassen. Wir wollen rechnen

R n , n = 1 ( 2 π ) 2 EIN d x d j 1 cos ( n ( x + j ) ) 2 cos ( x ) cos ( j ) = 1 2 ( 2 π ) 2 EIN d x d j 1 cos ( n ( x + j ) ) 1 cos ( x + j 2 ) cos ( x j 2 )

wo EIN ist das Quadrat 0 x , j 2 π .

Da der Integrand periodisch ist, kann der Definitionsbereich geändert werden EIN zu EIN ' so:

Rechtecke A und A'

Dann ändern Sie die Variablen zu

a = x + j 2 b = x j 2 d x d j = 2 d a d b

das Integral wird

R n , n = 1 ( 2 π ) 2 0 π d a π π d b 1 cos ( 2 n a ) 1 cos ( a ) cos ( b )

Das b Integral kann mit der Halb-Tan-Substitution durchgeführt werden

t = bräunen ( b / 2 ) cos ( b ) = 1 t 2 1 + t 2 d b = 2 1 + t 2 d t

geben

R n , n = 1 2 π 0 π d a 1 cos ( 2 n a ) Sünde ( a )

Die trigonometrische Identität

1 cos ( 2 n a ) = 2 Sünde ( a ) ( Sünde ( a ) + Sünde ( 3 a ) + + Sünde ( ( 2 n 1 ) a ) )

reduziert den Rest a integral zu

R n , n = 2 π ( 1 + 1 3 + + 1 2 n 1 )

Eine Wiederholungsbeziehung

Die restlichen Widerstände lassen sich nämlich ohne weitere Integrale bestimmen! Alles, was wir brauchen, ist Rotations-/Reflexionssymmetrie,

R n , m = R ± n , ± m = R ± m , ± n

zusammen mit der Wiederholungsrelation

R n + 1 , m + R n 1 , m + R n , m + 1 + R n , m 1 4 R n , m = 2 δ ( n , m )

was folgt aus R n = 2 G n und (2) . Es besagt, dass wir, wenn wir alle Widerstände außer einem in einer „Plus“-Form kennen, den fehlenden bestimmen können.

Beginnen Sie mit der trivialen Aussage that

R 0 , 0 = 0

Anwenden der Wiederholungsrelation at ( n , m ) = ( 0 , 0 ) und die Verwendung von Symmetrie gibt

R 1 , 0 = R 0 , 1 = 1 / 2

Die nächste Diagonale wird so gemacht:

Füllen Sie R11, dann R02 und R20 aus

Hier bedeutet das türkisfarbene Quadrat, dass wir ausfüllen R 1 , 1 mit der Formel für R n , n . Die gelben Quadrate zeigen eine Anwendung der zu bestimmenden Wiederholungsbeziehung an R 2 , 0 und R 0 , 2 . Die gepunkteten Quadrate zeigen auch Widerstände an, die wir im vorherigen Schritt durch Symmetrie bestimmen mussten.

Die Diagonale danach wird ähnlich ausgeführt, aber ohne dass die Formel für aufgerufen werden muss R n , n :

Geben Sie R12 und R21 ein, dann R03 und R30

Wiederholtes Abwechseln der beiden obigen Schritte ergibt einen Algorithmus zum Bestimmen von jedem R m , n . Offensichtlich sind alle von der Form

a + b / π

wo a und b sind rationale Zahlen. Jetzt kann dieser Algorithmus leicht von Hand ausgeführt werden, aber man könnte ihn genauso gut in Python codieren:

import numpy as np
import fractions as fr

N = 4
arr = np.empty((N * 2 + 1, N * 2 + 1, 2), dtype='object')

def plus(i, j):
    arr[i + 1, j] = 4 * arr[i, j] - arr[i - 1, j] - arr[i, j + 1] - arr[i, abs(j - 1)]

def even(i):
    arr[i, i] = arr[i - 1, i - 1] + [0, fr.Fraction(2, 2 * i - 1)]
    for k in range(1, i + 1): plus(i + k - 1, i - k)

def odd(i):
    arr[i + 1, i] = 2 * arr[i, i] - arr[i, i - 1]
    for k in range(1, i + 1): plus(i + k, i - k)

arr[0, 0] = 0
arr[1, 0] = [fr.Fraction(1, 2), 0]

for i in range(1, N):
    even(i)
    odd(i)

even(N)

for i in range(0, N + 1):
    for j in range(0, N + 1):
        a, b = arr[max(i, j), min(i, j)]
        print('(', a, ')+(', b, ')/π', sep='', end='\t')
    print()

Dies erzeugt die Ausgabe

40 368 3 π 80 π 49 2 6 236 fünfzehn π 24 5 π 1 2 352 105 π 17 2 24 π 46 3 π 4 1 2 + 4 3 π 46 fünfzehn π 24 5 π 1 2 2 4 π 4 π 1 2 8 3 π 1 2 + 4 3 π 6 236 fünfzehn π 1 2 2 π 4 π 1 2 46 3 π 4 80 π 49 2 0 1 2 2 4 π 17 2 24 π 40 368 3 π

woraus wir die endgültige Antwort ablesen können,

R 2 , 1 = 4 π 1 2

Es ist sehr interessant, dass derselbe Formalismus verwendet werden kann, um die Zweipunktfunktion eines masselosen Skalarfeldes auf einem Gitter zu lösen
du hast die Einheiten vergessen